LSAT and Law School Admissions Forum

Get expert LSAT preparation and law school admissions advice from PowerScore Test Preparation.

 Administrator
PowerScore Staff
  • PowerScore Staff
  • Posts: 8916
  • Joined: Feb 02, 2011
|
#40614
Complete Question Explanation
(The complete setup for this game can be found here: lsat/viewtopic.php?t=13252)

The correct answer choice is (C)

In this game, the typical situation is that most students create a setup that is very similar to the one presented during the setup discussion. They then arrive at this question, and are at an immediate loss since they do not have a Not Law on the fourth position that corresponds to one of the answer choices. While we could have explained the inference that solves this question during the game setup, the vast majority of students do not see this inference during the setup, and it is quite difficult to see overall. Thus, let’s address this question in the context most people encounter it: without the needed inference in hand when they arrive at the question and five answer choices.
When your main setup does not contain the information to answer a Global question immediately, your first step should be to examine prior work. Question #1 contains a complete solution to the game, and S was written fourth in that solution (this was also the case in question #3). Thus, answer choice (E) is eliminated. However, four answers still remain in contention, so what is the next step?
If no clear path to the correct answer seems obvious, then one approach would be to jump to question #5 and solve that question. Perhaps in solving that question an answer to #4 might become apparent, or perhaps at least one more answer could be eliminated in #4 (as it turns out, question #5 allows for both F and P to have been written fourth).
If you don’t take that approach, consider the remaining variables, and their effect on the setup:
Answer choice (A): This answer choice seems unlikely to be correct. In the setup, we identified certain restrictions that would occur when F was written fourth, but those restrictions did not seem to preclude a complete solution from being formed.
Answer choices (B) and (D): These two answers are similar since they each involve half of the GP block. As discussed elsewhere, when two answers seem functionally identical, they are not often the correct answer. While these two are not functionally identical, they are very similar, and would not seem to be the best starting point of attack.
Answer choice (C): This is the remaining answer choice, and would be the best first candidate to explore further because H affects several other variables (F and S) and because H also cannot be placed fifth, the space next to the one under consideration.
In answer choice (C), H—the middle variable in the F :longline: H :longline: S sequence—is placed in the middle position of the diagram. This effectively divides the spaces on the left and right of H into two equal-sized groups of three spaces each. Because of the third and fourth rules, there is no room for the GP block:
PT69_Game_#1_#4_diagram 1.png
Because when H was written fourth there are not two consecutive spaces available for GP, therefore H cannot have been written fourth, and answer choice (C) is correct.
You do not have the required permissions to view the files attached to this post.

Get the most out of your LSAT Prep Plus subscription.

Analyze and track your performance with our Testing and Analytics Package.